Difference between revisions of "Aufgaben:Exercise 3.8: Once more Mutual Information"

From LNTwww
Line 59: Line 59:
 
===Solution===
 
===Solution===
 
{{ML-Kopf}}
 
{{ML-Kopf}}
'''(1)'''&nbsp; The<u>correct solutions are 1 and 2</u>:
+
'''(1)'''&nbsp; The <u>correct solutions are 1 and 2</u>:
 
*With&nbsp; $X = \{0,\ 1,\ 2\}$,&nbsp; $Y = \{0,\ 1,\ 2\}$&nbsp;, &nbsp; $X + Y = \{0,\ 1,\ 2,\ 3,\ 4\}$ holds.&nbsp;  
 
*With&nbsp; $X = \{0,\ 1,\ 2\}$,&nbsp; $Y = \{0,\ 1,\ 2\}$&nbsp;, &nbsp; $X + Y = \{0,\ 1,\ 2,\ 3,\ 4\}$ holds.&nbsp;  
 
*The probabilities also agree with the given probability function.
 
*The probabilities also agree with the given probability function.
*Checking the other two specifications shows that $W = X Y + 2$ is also possible, but not $W = Y X + 2$.
+
*Checking the other two specifications shows that&nbsp; $W = X - Y + 2$&nbsp; is also possible, but not&nbsp; $W = Y - X + 2$.
  
 +
 +
[[File:P_ID2769__Inf_A_3_7d.png|right|frame|To calculate the mutual information]]
  
  
'''(2)'''&nbsp; From the 2D probability function&nbsp; $P_{ XW }(X, W)$&nbsp; on the specification page, one obtains for
+
'''(2)'''&nbsp; From the two-dimensional probability mass function&nbsp; $P_{ XW }(X, W)$&nbsp; on the specification page, one obtains for
 
*the joint entropy:
 
*the joint entropy:
 
:$$H(XW) =  {\rm log}_2 \hspace{0.1cm} (9)  
 
:$$H(XW) =  {\rm log}_2 \hspace{0.1cm} (9)  
Line 73: Line 75:
 
* the probability function of the random variable&nbsp; $W$:
 
* the probability function of the random variable&nbsp; $W$:
 
:$$P_W(W) = \big [\hspace{0.05cm}1/9\hspace{0.05cm}, \hspace{0.15cm} 2/9\hspace{0.05cm},\hspace{0.15cm} 3/9 \hspace{0.05cm}, \hspace{0.15cm} 2/9\hspace{0.05cm}, \hspace{0.15cm} 1/9\hspace{0.05cm} \big ]\hspace{0.05cm},$$
 
:$$P_W(W) = \big [\hspace{0.05cm}1/9\hspace{0.05cm}, \hspace{0.15cm} 2/9\hspace{0.05cm},\hspace{0.15cm} 3/9 \hspace{0.05cm}, \hspace{0.15cm} 2/9\hspace{0.05cm}, \hspace{0.15cm} 1/9\hspace{0.05cm} \big ]\hspace{0.05cm},$$
*the entropy of the random variable $W$:
+
*the entropy of the random variable&nbsp; $W$:
 
:$$H(W) = 2 \cdot \frac{1}{9} \cdot {\rm log}_2 \hspace{0.1cm} \frac{9}{1} + 2 \cdot \frac{2}{9} \cdot {\rm log}_2 \hspace{0.1cm} \frac{9}{2} +
 
:$$H(W) = 2 \cdot \frac{1}{9} \cdot {\rm log}_2 \hspace{0.1cm} \frac{9}{1} + 2 \cdot \frac{2}{9} \cdot {\rm log}_2 \hspace{0.1cm} \frac{9}{2} +
 
\frac{3}{9} \cdot {\rm log}_2 \hspace{0.1cm} \frac{9}{3}
 
\frac{3}{9} \cdot {\rm log}_2 \hspace{0.1cm} \frac{9}{3}
 
  {= 2.197\ {\rm (bit)}} \hspace{0.05cm}.$$
 
  {= 2.197\ {\rm (bit)}} \hspace{0.05cm}.$$
  
Thus, with&nbsp; $H(X) = 1.585 \ \rm bit$&nbsp; (was given), the result for the&nbsp; ''Mutual Information'':  
+
Thus, with&nbsp; $H(X) = 1.585 \ \rm bit$&nbsp; (was given), the result for the&nbsp; mutual information:  
 
:$$I(X;W) = H(X) + H(W) - H(XW) = 1.585 + 2.197- 3.170\hspace{0.15cm} \underline {= 0.612\ {\rm (bit)}} \hspace{0.05cm}.$$
 
:$$I(X;W) = H(X) + H(W) - H(XW) = 1.585 + 2.197- 3.170\hspace{0.15cm} \underline {= 0.612\ {\rm (bit)}} \hspace{0.05cm}.$$
  
[[File:P_ID2769__Inf_A_3_7d.png|right|frame|To calculate the mutual information]]
 
 
The left of the two diagrams illustrates the calculation of the mutual information&nbsp; $I(X; W)$&nbsp; between the first component&nbsp; $X$&nbsp; and the sum&nbsp; $W$.
 
The left of the two diagrams illustrates the calculation of the mutual information&nbsp; $I(X; W)$&nbsp; between the first component&nbsp; $X$&nbsp; and the sum&nbsp; $W$.
<br clear=all>
 
[[File:P_ID2770__Inf_A_3_7c.png|right|Joint probability between&nbsp; $Z$&nbsp; and&nbsp; $W$]]
 
'''(3)'''&nbsp;  The second graph shows the joint probability&nbsp; $P_{ ZW }(⋅)$.&nbsp; The scheme consists of&nbsp; $5 · 9 = 45$&nbsp; fields in contrast to the plot of&nbsp; $P_{ XW }(⋅)$&nbsp; on the data page with&nbsp; $3 · 9 = 27$&nbsp; fields.
 
*However, of the&nbsp; $45$&nbsp; fields, only nine are also assigned non-zero probabilities.&nbsp; The following applies to the compound entropy:  &nbsp; $H(ZW)  = 3.170\ {\rm (bit)} \hspace{0.05cm}.$
 
*With the further entropies&nbsp; $H(Z)  = 3.170\ {\rm (bit)}\hspace{0.05cm}$&nbsp; and&nbsp; $H(W)  = 2.197\ {\rm (bit)}\hspace{0.05cm}$&nbsp; according&nbsp; [[Aufgaben:3.07Z_Tupel_aus_tern%C3%A4ren_Zufallsgr%C3%B6%C3%9Fen| Exercise 3.8Z]]&nbsp; or the subquestion&nbsp; '''(2)'''&nbsp; of this exercise, one obtains for the mutual information:
 
:$$I(Z;W) = H(Z) + H(W) - H(ZW) \hspace{0.15cm} \underline {= 2.197\,{\rm (bit)}} \hspace{0.05cm}.$$
 
  
  
'''(4)'''&nbsp; <u>All three statements</u> are true, as can also be seen from the right-hand side of the two upper diagrams.
 
  
We attempt an interpretation of these numerical results:
+
[[File:P_ID2770__Inf_A_3_7c.png|right|frame|Joint probability between&nbsp; $Z$&nbsp; and&nbsp; $W$]]
* The joint probability&nbsp; $P_{ ZW }(⋅)$&nbsp;, like&nbsp; $P_{ XW }(⋅)$&nbsp;, is composed of nine equally probable elements unequal to 0. It is thus obvious that the compound entropies are also equal &nbsp; ⇒ &nbsp; $H(ZW) =  H(XW) = 3.170 \ \rm (bit)$.   
+
'''(3)'''&nbsp;  The second graph shows the joint probability&nbsp; $P_{ ZW }(⋅)$.&nbsp;
* If I know the tuple&nbsp; $Z = (X, Y)$&nbsp;, I naturally also know the sum&nbsp; $W = X + Y$.&nbsp; Thus&nbsp; $H(W|Z) = 0$.  
+
*The scheme consists of&nbsp; $5 · 9 = 45$&nbsp; fields in contrast to the plot of&nbsp; $P_{ XW }(⋅)$&nbsp; on the data page with&nbsp; $3 · 9 = 27$&nbsp; fields.
 +
*However, of the&nbsp; $45$&nbsp; fields, only nine are also assigned non-zero probabilities.&nbsp; The following applies to the joint entropy:  &nbsp; $H(ZW)  = 3.170\ {\rm (bit)} \hspace{0.05cm}.$
 +
*With the further entropies&nbsp; $H(Z)  = 3.170\ {\rm (bit)}\hspace{0.05cm}$&nbsp; and&nbsp; $H(W)  = 2.197\ {\rm (bit)}\hspace{0.05cm}$&nbsp; according to&nbsp; [[Aufgaben:Exercise_3.8Z:_Tuples_from_Ternary_Random_Variables| Exercise 3.8Z]]&nbsp; or the subquestion&nbsp; '''(2)'''&nbsp; of this exercise, one obtains for the mutual information:
 +
:$$I(Z;W) = H(Z) + H(W) - H(ZW) \hspace{0.15cm} \underline {= 2.197\,{\rm (bit)}} \hspace{0.05cm}.$$
 +
<br clear=all>
 +
'''(4)'''&nbsp; <u>All three statements</u> are true, as can also be seen from the right-hand side of the two upper diagrams.&nbsp; We attempt an interpretation of these numerical results:
 +
* The joint probability&nbsp; $P_{ ZW }(⋅)$&nbsp;is composed, like&nbsp; $P_{ XW }(⋅)$&nbsp;, of nine equally probable elements unequal to 0.&nbsp; It is thus obvious that the joint entropies are also equal &nbsp; ⇒ &nbsp; $H(ZW) =  H(XW) = 3.170 \ \rm (bit)$.   
 +
* If I know the tuple&nbsp; $Z = (X, Y)$,&nbsp; I naturally also know the sum&nbsp; $W = X + Y$.&nbsp; Thus&nbsp; $H(W|Z) = 0$.  
 
*In contrast,&nbsp; $H(Z|W) \ne 0$.&nbsp; Rather,&nbsp; $H(Z|W) = H(X|W) = 0.973  \ \rm (bit)$.
 
*In contrast,&nbsp; $H(Z|W) \ne 0$.&nbsp; Rather,&nbsp; $H(Z|W) = H(X|W) = 0.973  \ \rm (bit)$.
 
*The random variable&nbsp; $W$&nbsp; thus provides exactly the same information with regard to the tuple&nbsp; $Z$&nbsp; as for the individual component&nbsp; $X$.&nbsp; This is the verbal interpretation of the statement&nbsp; $H(Z|W) = H(X|W)$.
 
*The random variable&nbsp; $W$&nbsp; thus provides exactly the same information with regard to the tuple&nbsp; $Z$&nbsp; as for the individual component&nbsp; $X$.&nbsp; This is the verbal interpretation of the statement&nbsp; $H(Z|W) = H(X|W)$.
*The joint information of&nbsp; $Z$&nbsp; and&nbsp; $W$&nbsp; &nbsp; ⇒ &nbsp; $I(Z; W)$&nbsp; is greater than the joint information of&nbsp; $X$&nbsp; and&nbsp; $W$  &nbsp; ⇒ &nbsp; $I(X; W)$, because&nbsp; $H(W|Z) =0$&nbsp;, while&nbsp; $H(W|X)$&nbsp; is non-zero, namely exactly as great as&nbsp; $H(X)$ :
+
*The joint information of&nbsp; $Z$&nbsp; and&nbsp; $W$&nbsp; &nbsp; ⇒ &nbsp; $I(Z; W)$&nbsp; is greater than the joint information of&nbsp; $X$&nbsp; and&nbsp; $W$  &nbsp; ⇒ &nbsp; $I(X; W)$,&nbsp; because&nbsp; $H(W|Z) =0$,&nbsp; while&nbsp; $H(W|X)$&nbsp; is non-zero, namely exactly as great as&nbsp; $H(X)$ :
 
:$$I(Z;W)  = H(W) - H(W|Z) = 2.197 - 0= 2.197\,{\rm (bit)} \hspace{0.05cm},$$
 
:$$I(Z;W)  = H(W) - H(W|Z) = 2.197 - 0= 2.197\,{\rm (bit)} \hspace{0.05cm},$$
 
:$$I(X;W) =  H(W) - H(W|X) = 2.197 - 1.585= 0.612\,{\rm (bit)} \hspace{0.05cm}.$$
 
:$$I(X;W) =  H(W) - H(W|X) = 2.197 - 1.585= 0.612\,{\rm (bit)} \hspace{0.05cm}.$$

Revision as of 18:28, 20 September 2021

2D–Functions 
 $P_{ XY }$  und  $P_{ XW }$

We consider the tuple  $Z = (X, Y)$, where the individual components  $X$  and  $Y$  each represent ternary random variables:

$$X = \{ 0 ,\ 1 ,\ 2 \} , \hspace{0.3cm}Y= \{ 0 ,\ 1 ,\ 2 \}.$$

The joint probability function  $P_{ XY }(X, Y)$  of both random variables is given in the upper graph. 

In  Exercise 3.8Z  this constellation is analysed in detail.  One obtains as a result (all data in "bit"):

  • $H(X) = H(Y) = \log_2 (3) = 1.585,$
  • $H(XY) = \log_2 (9) = 3.170,$
  • $I(X, Y) = 0,$
  • $H(Z) = H(XZ) = 3.170,$
  • $I(X, Z) = 1.585.$

Furthermore, we consider the random variable  $W = \{ 0,\ 1,\ 2,\ 3,\ 4 \}$, whose properties result from the composite probability function  $P_{ XW }(X, W)$  according to the sketch below.  The probabilities are zero in all fields with a white background.

What is sought in the present exercise is the mutual information between

  • the random variables  $X$  and  $W$   ⇒   $I(X; W)$,
  • the random variables  $Z$  and  $W   ⇒   I(Z; W)$.



Hints:


Questions

1

How might the variables  $X$,  $Y$  and  $W$  be related?

$W = X + Y$,
$W = X - Y + 2$,
$W = Y - X + 2$.

2

What is the mutual information between the random variables  $X$  and  $W$?

$I(X; W) \ = \ $

$\ \rm bit$

3

What is the mutual information between the random variables  $Z$  and  $W$?

$I(Z; W) \ = \ $

$\ \rm bit$

4

Which of the following statements are true?

  $H(ZW) = H(XW)$  is true.
  $H(W|Z) = 0$  is true.
  $I(Z; W) > I(X; W)$  is true.


Solution

(1)  The correct solutions are 1 and 2:

  • With  $X = \{0,\ 1,\ 2\}$,  $Y = \{0,\ 1,\ 2\}$ ,   $X + Y = \{0,\ 1,\ 2,\ 3,\ 4\}$ holds. 
  • The probabilities also agree with the given probability function.
  • Checking the other two specifications shows that  $W = X - Y + 2$  is also possible, but not  $W = Y - X + 2$.


To calculate the mutual information


(2)  From the two-dimensional probability mass function  $P_{ XW }(X, W)$  on the specification page, one obtains for

  • the joint entropy:
$$H(XW) = {\rm log}_2 \hspace{0.1cm} (9) = 3.170\ {\rm (bit)} \hspace{0.05cm},$$
  • the probability function of the random variable  $W$:
$$P_W(W) = \big [\hspace{0.05cm}1/9\hspace{0.05cm}, \hspace{0.15cm} 2/9\hspace{0.05cm},\hspace{0.15cm} 3/9 \hspace{0.05cm}, \hspace{0.15cm} 2/9\hspace{0.05cm}, \hspace{0.15cm} 1/9\hspace{0.05cm} \big ]\hspace{0.05cm},$$
  • the entropy of the random variable  $W$:
$$H(W) = 2 \cdot \frac{1}{9} \cdot {\rm log}_2 \hspace{0.1cm} \frac{9}{1} + 2 \cdot \frac{2}{9} \cdot {\rm log}_2 \hspace{0.1cm} \frac{9}{2} + \frac{3}{9} \cdot {\rm log}_2 \hspace{0.1cm} \frac{9}{3} {= 2.197\ {\rm (bit)}} \hspace{0.05cm}.$$

Thus, with  $H(X) = 1.585 \ \rm bit$  (was given), the result for the  mutual information:

$$I(X;W) = H(X) + H(W) - H(XW) = 1.585 + 2.197- 3.170\hspace{0.15cm} \underline {= 0.612\ {\rm (bit)}} \hspace{0.05cm}.$$

The left of the two diagrams illustrates the calculation of the mutual information  $I(X; W)$  between the first component  $X$  and the sum  $W$.


Joint probability between  $Z$  and  $W$

(3)  The second graph shows the joint probability  $P_{ ZW }(⋅)$. 

  • The scheme consists of  $5 · 9 = 45$  fields in contrast to the plot of  $P_{ XW }(⋅)$  on the data page with  $3 · 9 = 27$  fields.
  • However, of the  $45$  fields, only nine are also assigned non-zero probabilities.  The following applies to the joint entropy:   $H(ZW) = 3.170\ {\rm (bit)} \hspace{0.05cm}.$
  • With the further entropies  $H(Z) = 3.170\ {\rm (bit)}\hspace{0.05cm}$  and  $H(W) = 2.197\ {\rm (bit)}\hspace{0.05cm}$  according to  Exercise 3.8Z  or the subquestion  (2)  of this exercise, one obtains for the mutual information:
$$I(Z;W) = H(Z) + H(W) - H(ZW) \hspace{0.15cm} \underline {= 2.197\,{\rm (bit)}} \hspace{0.05cm}.$$


(4)  All three statements are true, as can also be seen from the right-hand side of the two upper diagrams.  We attempt an interpretation of these numerical results:

  • The joint probability  $P_{ ZW }(⋅)$ is composed, like  $P_{ XW }(⋅)$ , of nine equally probable elements unequal to 0.  It is thus obvious that the joint entropies are also equal   ⇒   $H(ZW) = H(XW) = 3.170 \ \rm (bit)$.
  • If I know the tuple  $Z = (X, Y)$,  I naturally also know the sum  $W = X + Y$.  Thus  $H(W|Z) = 0$.
  • In contrast,  $H(Z|W) \ne 0$.  Rather,  $H(Z|W) = H(X|W) = 0.973 \ \rm (bit)$.
  • The random variable  $W$  thus provides exactly the same information with regard to the tuple  $Z$  as for the individual component  $X$.  This is the verbal interpretation of the statement  $H(Z|W) = H(X|W)$.
  • The joint information of  $Z$  and  $W$    ⇒   $I(Z; W)$  is greater than the joint information of  $X$  and  $W$   ⇒   $I(X; W)$,  because  $H(W|Z) =0$,  while  $H(W|X)$  is non-zero, namely exactly as great as  $H(X)$ :
$$I(Z;W) = H(W) - H(W|Z) = 2.197 - 0= 2.197\,{\rm (bit)} \hspace{0.05cm},$$
$$I(X;W) = H(W) - H(W|X) = 2.197 - 1.585= 0.612\,{\rm (bit)} \hspace{0.05cm}.$$